3
$\begingroup$

I heard that Zorn's lemma is equivalent to Tukey's lemma. Now I've proved that Zorn's lemma implies that Tukey's lemma but I cannot prove that Tukey's lemma implies Zorn's lemma. How to show this?

  • 0
    This question is somewhat related: [Tukey's lemma by axiom of choice](http://math.stackexchange.com/questions/135655/tukeys-lemma-by-axiom-of-choice).2012-05-31

2 Answers 2